Shortcuts to find a solution to a double integral

Click For Summary
The discussion focuses on finding shortcuts to evaluate a double integral, specifically seeking to leverage symmetries or properties of odd functions. A participant points out that the original boundaries for the integral were incorrect, suggesting a need to redefine the integration region. The correct boundaries include two regions: one for a right-angled triangle and another smaller triangle. The initial approach involved calculating the integral over a larger triangle and then subtracting the integral of a smaller region. The conversation emphasizes the importance of accurately defining integration boundaries to simplify the evaluation process.
Amaelle
Messages
309
Reaction score
54
Homework Statement
Let T be the triangle of vertices (0,0),(2,2),(-4,0) . compute the integral ∫∫xydxdy over the region T
Relevant Equations
0<=y<=(x+4)/3
-4<=x<=2
I know the value of this integral is equal to 0, but I would like to see if there is any tricks to spot this answer using symmetries or even odd propreties?
Thanks in advance
 
Physics news on Phys.org
This isn't a trick but I thought I'd just point out that the boundaries that you gave aren't right (i.e. yours is for a right angled triangle); you need to split the region of integration into ##0 \leq y \leq \frac{x+4}{3}##, ##-4 \leq x \leq 0## as well as ##x \leq y \leq \frac{x+4}{3}##, ##0 \leq x \leq 2##.
 
  • Like
Likes Amaelle
yes thanks a lot , you are right the point is that I used a big a triangle with the following boundaries ( and calculated the integral over it) and then substructed the inetgral over a s smaller integral with the following boundaries
0<x<2 and 0<y<x
 
  • Like
Likes etotheipi
Question: A clock's minute hand has length 4 and its hour hand has length 3. What is the distance between the tips at the moment when it is increasing most rapidly?(Putnam Exam Question) Answer: Making assumption that both the hands moves at constant angular velocities, the answer is ## \sqrt{7} .## But don't you think this assumption is somewhat doubtful and wrong?

Similar threads

Replies
10
Views
2K
  • · Replies 17 ·
Replies
17
Views
4K
Replies
4
Views
2K
  • · Replies 4 ·
Replies
4
Views
2K
  • · Replies 5 ·
Replies
5
Views
2K
  • · Replies 6 ·
Replies
6
Views
1K
Replies
3
Views
2K
  • · Replies 4 ·
Replies
4
Views
1K
  • · Replies 2 ·
Replies
2
Views
1K
Replies
5
Views
2K